ST5 Sector and Stock Profit

Discussion in 'SP5' started by William Edwards, Sep 21, 2016.

  1. Hi,

    I am struggling to split the sector and stock profit on the April 2015and April 2016 exams. It is question5 on both papers.

    Is there a formula I am missing as I think I am using the one in the notes but I just cannot seem to get the right answers.

    Kind regards,

    Will
     
  2. Simon James

    Simon James ActEd Tutor Staff Member

    Hi Will - you should be able to get there using the approach in the Notes - I'll need a bit more info to help you out. Can you show what you have done/your results?
     
  3. Hi Simon,

    In the April 2015 paper I cannot seem to get the stock and sector profits unsing the formula. So as an example I can get the stock selection profit for the different groups using the formula Faa - Fan = x= 7.28. I.e 88.155 - 80.8355 = 7.28%. But I cannot seem to work out the stock profit using the Fan - Fnn. Then when I tried (actual weight - notional weight) * (sector notional return - benchmark return) I also couln't get it. Does it have to be the benchmark and notional retrn for that explicit time period?

    And on the 2016 paper I cannot work it out as they only give you one set of returns. Therefore the notional and actual are the same and hence I cannot work out either the stock profit or the sector profit.

    Does that help?

    Kind regards,

    Will
     
  4. Simon James

    Simon James ActEd Tutor Staff Member

    Hi

    You should be using the returns over the three years (so 48.75% for the benchmark fund return).

    For 2015, the stock selection for domestic equities is:
    \( 0.25 \times (35.20\% - 38.72\% ) = - 0.88\% \)
    and sector selection is
    \( (0.25 - 0.40) \times (38.72\% - 48.75\% ) = + 1.50\% \)

    For 2016, we have to work at sector level - consider the benchmark allocations/returns and actual allocations/returns for each sector (eg equities is A & B & C combined). If we carried on working stock by stock, then the actual and benchmark returns would be equal and we would have no stock selection profits. Have a go and let me know if you are still struggling.

    Simon
     
  5. Hi Simon,

    Thanks for the help.

    Completely understand the April 2015 paper.

    On the April 2016 paper is the sector selection profits just the relative performance - stock selection?

    I cannot seem to get the answer using the above equations so for example for period 2 and stock selection I have :

    Stock selection - 0.35 x (15.7-16.67) + 0.30 x (3-3.7) + .30 x (16.67 - 23.33) = -2.55%

    Kind regards,

    Will
     
  6. Simon James

    Simon James ActEd Tutor Staff Member

    Hi Will

    Are you sure about the "3" in the middle of that expression (the FI actual returns) - I have 3.333 which I think gives the correct answer?

    Simon
     
  7. JoshN

    JoshN Member

    Hi,

    I am having a bit of trouble understanding the solution to September 2013 Q8 part c. I don't understand the “Average” sector weighting x sector benchmark performance calculation done for Period 2. The examiners report seems to use the sector portfolio weights (30%,30%,40%) and the stock portfolio weights (35%, 25% and 40% after rebalance) and average them, then multiplies by the benchmark return. I understand that this would give a notional portfolio using actual weightings with bench returns that can be compared to the actual portfolio to get stock selection profits. However, surely the "actual" weights cannot be calculated in this way as the stock and sector portfolios have different values by period 2 (stock=111m, sector=113m) so does this not need to be reflected when calculating the actual weights at time 2? Was this just ignored due to the small difference of 2m in the two portfolio values?

    Sorry if my question is unclear.

    Thanks,
     
  8. Simon James

    Simon James ActEd Tutor Staff Member

    Hi Josh. The portfolios are rebalanced at the start of each period - does this solve the issue?

    Simon
     
  9. JoshN

    JoshN Member

    Hi Simon.

    Sorry, no this doesn't solve my issue. As I understand things:
    Stock profits = actual weightings, actual stock returns - actual weightings, bench stock returns. The below part of the solution seems to be calculating the value of the "actual weightings, bench stock returns" portfolio:

    From the above part of the solution, I do not understand how the weightings for period 2 were derived, as it seems to imply the fund value=200 but at this stage the actual fund values were 111m for stock manager, 113m for sector manager. Therefore, after rebalanacing, shouldn't the actual weights in each sector be:
    I = (30%*113+35%*111)/224
    U = (30%*113+25%*111)/224
    F= (40*113+40%*111)/224
    ?
     
  10. Simon James

    Simon James ActEd Tutor Staff Member

    Ah, ok. I see what you mean. You are right the Period 2 returns should allow for the Period 1. This is what we did in the ActEd ASET. The weights are 32.4777%, 27.5223%, 40.0000%. This results in a notional fund return of 8.42% (rather than 8.43%). The minor differences in the end of period 1 positions fortunately don't have too much impact.
     
  11. JoshN

    JoshN Member

    Great, thanks; I still struggle with performance attribution so wanted to make sure I wasn't going in the wrong direction with my answer, even though differences are minor.

    Is it also usually best to take the sector selection profits as the balancing item in the exam? Obviously this causes issues if the stock profits had been incorrectly calculated so how harshly would this be penalised (if anyone knows?)
     
    Simon James likes this.
  12. Simon James

    Simon James ActEd Tutor Staff Member

    Sometimes, yes, you can simply take sector selection as the balancing item. If you have made an earlier error, you should (if you show your working) get follow-thru marks :)
     
  13. Following up on William Edwards post above on the April 2016 performance. I have been able to get the stock performance ok but I am having issues getting the sector performance. I know I can just take the stock performance away from the outperformance but the actual calculation is not tying back for me when i calculate it itself. The formula I am using is (Actual % allocation - benchmark % Allocation) x (benchmark return of asset - benchmark return of fund).
    So for Year 2 I have the following formula:
    (0.35 - 0.3) x (16.67% - 13.1%) + (0.3 - 0.3) x (3.68% - 13.1%) + (0.3 - 0.3) x (23.3% - 13.1%) which gives 0.179% rather than the expected answer of 0.83% (outperformance of-1.60% - stock selection performance of -2.43%)
     
  14. Simon James

    Simon James ActEd Tutor Staff Member

    Hi Actuaryin training - have you ignored the cash in that formula?
     
  15. Yes - thank you! When i saw the 0% benchmark return I assumed (incorrectly) that that was zero.
     

Share This Page